Question 24

Read the following statement:

A manager seeks approval for conducting a training programme on 'openness'. He puts forward the following arguments in favour of the program to his CEO.

Which of the following arguments is the least likely to have a logical fallacy?

Solution

Option A: Since a group of employees attended the program, a change in the openness of only one participant does not give much incentive to the CEO to give the approval. Thus, this is not the correct option.

Option B: Although this option does speak good about the programme, this does not give any consolidation to the efficacy of the program. Thus, this is not the correct option.

Option C: Since this option provides consolidation to the efficacy of the program, this is the correct answer.

Option D: Giving his CEO an ultimatum is not a good idea for the approval. Thus, this is not the correct option.

Option E: Emotional appeal will have a similar effect as that of the previous option. Thus, this is not the correct option.

Thus, the correct option is E.


Create a FREE account and get:

  • All Quant Formulas and shortcuts PDF
  • XAT previous papers with solutions PDF
  • XAT Trial Classes for FREE

    cracku

    Boost your Prep!

    Download App